Partícula cargada observada desde un marco de referencia inercial y no inercial

Una partícula cargada fijada a un marco. S está acelerando con un marco inercial S . Para un observador A en el S marco, la partícula cargada está acelerando (estando unida al marco S ) y por lo tanto, observa que irradia. Sin embargo, para el observador no inercial B que se encuentra en el marco no inercial S , la partícula cargada está en reposo y, por lo tanto, no irradia nada.

A infiere que la partícula cargada irradia pero B infiere que no. ¿Pueden ambas inferencias (mutuamente contradictorias) ser correctas simultáneamente? Si es así, ¿cómo?

Piensa, ¿B siente la aceleración? Si está de acuerdo en que sí, siente que B no está en un marco inercial.
@HolgerFiedler ¿Dije que B no siente ninguna aceleración? Hice una edición para enfatizar que B obviamente acelera, pero tanto B como la partícula cargada están unidas al marco. S , están en reposo entre sí.
SRS, el punto es que S ' no es un marco inercial y para tal marco una partícula irradiará.
no entiendo esto B no ve la partícula cargada para acelerar ni siquiera cambiar su posición. Si la partícula cargada no cambia de posición, ¿se puede decir que acelera?
" Sin embargo, para el observador no inercial B que se encuentra en el marco no inercial S′, la partícula cargada está en reposo y, por lo tanto, no irradia en absoluto ". - ¿Qué quiere decir con "la partícula irradia" en el marco inercial? ? ¿Y en marco no inercial? Si quiere decir que el campo EM cerca de la partícula es de un tipo especial, ¿qué tipo de campo implicaría "partículas radiadas"?

Respuestas (6)

Primero, señalaré que, a menos que el marco no inercial tenga una aceleración variable , existen algunas dudas sobre si irradia o no. https://en.wikipedia.org/wiki/Paradox_of_a_charge_in_a_gravitational_field

Suponiendo que hay una aceleración cambiante, sí, la partícula irradia, y esto se puede observar en ambos marcos. Verlo irradiar sería una forma de determinar que, de hecho, se encuentra en un marco no inercial, suponiendo que su oído interno no se lo haya dicho ya.

Un electrón que cae no irradia.
¿La radiación de frenado como se describe aquí en.wikipedia.org/wiki/Bremsstrahlung es consistente con su afirmación de que la aceleración debe cambiar para inducir la radiación?

La noción de una carga puntual ideal en realidad no está completamente bien definida en la E&M clásica debido a problemas con la energía propia infinita, la reacción inversa, etc., por lo que no hay una respuesta definitiva a esta pregunta. Vea las extensas discusiones en ¿Una partícula cargada en constante aceleración emite radiación EM o no? y ¿Una partícula cargada que se acelera en un campo gravitatorio irradia? .

Lo único en lo que todos estarán de acuerdo es en el momento adecuado d τ .

La energía no se conserva pero sí la acción, porque se expresa en términos de tiempo propio.

Para ver lo que se ve en el otro marco de referencia, se debe hacer una transformación de coordenadas de un vector de 4:

( V ) m = X m X v V v
o
( V ) m = X v X m V v

si las cosas dependen de pasar por múltiples referencias

( V ) m = X m X v ( V ) v
( V ) m = X m X v X v X σ V σ
. . .
Lo que uno podría ver como un desplazamiento hacia el rojo de algún grado debido al campo gravitatorio, otra persona no vería el desplazamiento hacia el rojo si estuviera en un curso directo hacia el generador del campo gravitatorio. Pero nuevamente, un tercer observador vería tanto el desplazamiento hacia el rojo del primer cuadro como el desplazamiento hacia el azul del segundo cuadro si estuviera entre los 2 "cuadros" anteriores de pie - "en reposo".

Entonces para el tensor electromagnético F m v ( = ϕ m X v ϕ v X m ) se debe realizar una transformación de coordenadas:

( F ) m v = X α X m X β X v F α β

Estas transformaciones se basan en el tiempo adecuado y expresan el "valor" real visto en el "otro" marco de referencia.

Si uno hace los cálculos para las derivadas parciales de la métrica de Minkowski, obtendrá la transformación de Lorentz de un impulso general que se puede aplicar al tensor electromagnético.

( F ) m v = Λ m α Λ v β F α β

Un cargo será influenciado de manera diferente ( d pags m d τ = mi F v m tu v ) por ( F ) m v que por F m v dependiendo del movimiento relativo del generador de F m v . El campo una vez generado es "independiente" de la fuente y también se debe usar la posición/velocidad "retrasada" de la fuente en el momento apropiado en el "pasado".

Nota: aquí F v m = η m σ F σ v

Uno podría detenerse por un momento y leer la Conferencia de Feynman (para una mejor descripción): http://www.feynmanlectures.caltech.edu/II_26.html

B no infiere que la carga no irradia. La carga en sí siente una reacción inversa debido a la radiación emitida, que se debe a la interacción que tiene con su propio campo electromagnético. Recientemente se dio un tratamiento riguroso de la fuerza propia electromagnética, ver aquí .

Los enfoques más antiguos conducían a inconsistencias, pero es una mala práctica esconderse detrás de estas inconsistencias para argumentar a favor de la mala física. Un punto de vista razonable de la física en ausencia de un tratamiento matemático riguroso siempre debería haber sido que la carga irradia en todos los marcos porque siempre se puede invocar algún efecto físico de la radiación emitida que es manifiestamente clara en todos los marcos. Uno puede invocar la conservación de la energía, poniendo la carga en un cohete y luego considerando una quema de cohete finita y luego argumentando que la energía de los fotones emitidos debe tenerse en cuenta.

Otro argumento de la física que no depende de las sutilezas matemáticas es invocar al observador C que viaja cerca de la velocidad de la luz, tan cerca que la radiación emitida es azul desplazada a rayos gamma duros y C es frito por estos rayos gamma. Entonces, ¿cómo explica B lo que le pasó a C?

La conservación de la energía es ambigua en el caso de partículas puntuales cargadas, porque la autoenergía de una carga es infinita, e infinito más cualquier número finito sigue siendo infinito.
@tparker Sí, pero este problema se puede abordar mediante un procedimiento de regularización adecuado, por ejemplo, como se usa en este artículo . Si considera un proceso en el que la carga se aceleró durante una duración finita, entonces la energía propia (regularizada), cualquiera que sea, debería ser la misma en el estado inicial y final.

Esto no es complicado.

  • La cuestión de si la partícula irradia debe resolverse en su marco de referencia.

  • La cuestión de si se observa la radiación debe resolverse en el marco de referencia del observador.

Cambiar el observador o agregar más o diferentes observadores no afecta si hay radiación o no.

La aceleración no se determina relativamente, puedes saber si estás en un marco acelerado sin referirte a un marco externo porque sientes la fuerza.

Una partícula cargada fijada a un marco S′ está acelerando frente a un marco inercial S. Para un observador A en el marco S, la partícula cargada está acelerando (estando unida al marco S′) y por lo tanto, observa que irradia.

Sí, lo observa irradiar, porque irradia. No olvides la naturaleza ondulatoria de la materia. O producción en pareja. Hiciste ese electrón (y un positrón) a partir de una onda electromagnética. Y tampoco olvides la dispersión de Compton :

ingrese la descripción de la imagen aquí Imagen cortesía de la hiperfísica de Rod Nave

Usted puede efectivamente "rebanar una rebanada" de la energía cinética del fotón incidente y dársela al electrón, con lo cual el electrón se mueve . El Compton inverso es lo mismo al revés. Bremsstrahlung no es tan diferente. El electrón es real, el fotón es real, la interacción es real, el movimiento es real.

Sin embargo, para el observador no inercial B que se encuentra en el marco no inercial S′, la partícula cargada está en reposo y, por lo tanto, no irradia en absoluto.

No tan. El observador sabe que no está en reposo porque puede sentir la aceleración. Está siguiendo el ritmo del electrón, por lo que sabe que tampoco está en reposo. Además, puede detectar ese fotón. El electrón es real, el fotón es real, la interacción es real, el movimiento es real. Lo que no es, es el marco. Eso es algo abstracto.

A infiere que la partícula cargada irradia pero B infiere que no. ¿Pueden ambas inferencias (mutuamente contradictorias) ser correctas simultáneamente?

No. La partícula acelerada irradia*. No dejes que las cosas abstractas que no existen nublen tu comprensión de las cosas que existen.

* Un electrón que cae no irradia. Pero esa es otra tetera de pescado por completo .